Q) Claim Counting (10.03.50a)

by admin on April 9, 2010 · 4 comments

in Exam Questions

Test takers report a #50  from the October 2003 (AM) patent bar exam is in the current question pool.

50. Ben hires a registered practitioner to prosecute his patent application. The practitioner drafted an application having fifteen claims. Claim 1 is independent, and each of claims 2-15 are singularly dependent upon claim 1. A proper non-final Office action is mailed to the practitioner. Following consultation with Ben, the practitioner timely prepared, signed, and filed a reply to the Office action containing an amendment that does not add new matter, but does add claims 16-27. Each of claims 16-27 is directed to the same invention sought to be patented through claims 1-15. The dependency of each of claims 16-27 reads “any of claims 5-15.” For purposes of fee calculation in accordance with the patent laws, rules and procedures as related in the MPEP, how many total claims are contained in the application after the amendment is entered?
(A) One hundred thirty-six.
(B) One hundred thirty-five.
(C) Twenty-seven.
(D) One hundred forty-seven.
(E) Fifteen.



ANSWER: (D) is the most correct answer. 37 CFR § 1.75; MPEP § 608.01(n). As explained in MPEP § 608.01(n), under the heading “Multiple Dependent Claims,” subheading “Acceptable Multiple Dependent Claim Wording” the multiple dependent claim wording of new claims 16-27 is proper. See, for example, “any one of the preceding claims,” and “in any of claims 1-3 or 7-9.” 37 CFR § 1.75(c) states “For fee calculation purposes under § 1.16, a multiple dependent claim will be considered to be that number of claims to which direct reference is made therein.” Therefore, claims 16-27 would each have a claim value of eleven and the total number of claims for fee calculation is one hundred forty-seven (12 x 11 = 132 + 15 = 147). Answers (A) and (B) are incorrect because they are not the correct total. Answer (C) is incorrect because the multiple dependent claims have not been calculated in accordance with 37 CFR § 1.75. Answer (E) is incorrect because the question asks for the total after the amendment adding claims 16-27 has been entered.

1 passedatlastNo Gravatar June 19, 2012 at 5:28 pm

Got this 6/19/12.

2 RamboNo Gravatar August 12, 2013 at 3:01 pm

I don’t understand how the calculation is done. Why is the 11 multiplied by the 12 ? I understand how to get the 11 (27-16) but why it is multiplied by 12 and why is the 15 added to it. Help! I take the test in a few days!

3 TonyNo Gravatar August 17, 2013 at 8:04 pm

claims 16 to 27 –> “12”
each of claims 16 to 27 depends on any of claims 5-15 –> “11”
“15” original claims

4 RamboNo Gravatar August 17, 2013 at 8:51 pm

Thanks,Tony! I just made the error of subtracting them instead of the method you showed. It is crystal clear now.

Previous post:

Next post: